Someone please help me fast !!

Someone Please Help Me Fast !!

Answers

Answer 1
The second answer is correct.
Answer 2
Second answer is the right one

Related Questions

Please answer this question now

Answers

Answer:

54

Step-by-step explanation:

To solve problems like this, always recall the "Two-Tangent theorem", which states that two tangents of a circle are congruent if they meet at an external point outside the circle.

The perimeter of the given triangle = IK + KM + MI

IK = IJ + JK = 13

KM = KL + LM = ?

MI = MN + NI ?

Let's find the length of each tangents.

NI = IJ = 5 (tangents from external point I)

JK = IK - IJ = 13 - 5 = 8

JK = KL = 8 (Tangents from external point K)

LM = MN = 14 (Tangents from external point M)

Thus,

IK = IJ + JK = 5 + 8 = 13

KM = KL + LM = 8 + 14 = 22

MI = MN + NI = 14 + 5 = 19

Perimeter = IK + KM + MI = 13 + 22 + 19 = 54

Please help me!! I'll give brainliest btw

Answers

Answer:

e. cannot be determined

Step-by-step explanation:

m = 1/2( x + x + 10) = 1/2(2x + 10) = x + 5

n = 1/2(x² + x + 10) =

m/n = (x+5)/(1/2(x² + x + 10))

Not enough information

+
If the
sides of a triangles are
6, 8 and n. how
many integer values of n
could be the
measure of the
third side of the triangle?

Answers

Answer:

11

Step-by-step explanation:

The sum of the shortest two sides must be greater than the longest side.

If n is the longest side:

6 + 8 > n

14 > n

If 8 is the longest side:

6 + n > 8

n > 2

So n must be an integer greater than 2 and less than 14.

n can be 3, 4, 5, 6, 7, 8, 9, 10, 11, 12, or 13.

There are 11 possible integers.

[tex] \LARGE{ \boxed{ \rm{ \purple{Answer}}}}[/tex]

We know,

Sum of two sides of a triangle > Third side

Then,

⇛ 6 + 8 > n

⇛ 14 > n

Nextly,

Difference of two sides of a triangle < Third side

Then,

⇛ 8 - 6 < n

⇛ 2 < n

Then, Range of third side:

☃️ 2 < n < 14

Possible measures of 3rd sides = 3, 4, 5, 6, 7, 8, 9, 10, 11, 12 or 13.

There are 11 possible values of 3rd side. Out of them, any measure is the length of 3rd side.

━━━━━━━━━━━━━━━━━━━━

Could anyone help me with number 25 THANK YOU!!!

Answers

Answer:

ΔABC ~ ΔQPR by the Angle-Angle (AA) similarity theorem of two triangles

Step-by-step explanation:

The coordinates of the vertices are given as follows;

A = (1, 2), B =(9, 8), C = (1, 8)

P= (5, -3), Q = (-7, 6), R = (-7, -3)

The given dimensions of AB and PQ are 10, and 15 respectively

The, l lengths of the sides of triangles are found as follows;

[tex]l = \sqrt{\left (y_{2}-y_{1} \right )^{2}+\left (x_{2}-x_{1} \right )^{2}}[/tex]

For segment BC, we have;

B =(9, 8), C = (1, 8)

(x₁, y₁) = (9, 8), (x₂, y₂) = (1, 8), substituting gives;

Length BC = 8

For length CA, C = (1, 8) A = (1, 2)

(x₁, y₁) = (1, 8)

(x₂, y₂) = (1, 2)

The length found by substituting the values for (x₁, y₁), (x₂, y₂) in the length equation gives; Length CA = 6

Given that length  CA² + BC² = 8² + 6² = 64 + 36 = 100 = BA², we have by Pythagoras theorem, we have ΔABC is a right triangle

Similarly, for ΔQPR, we have;

Length QR, Q = (-7, 6), R = (-7, -3) = 9

Length PR, P= (5, -3), R = (-7, -3) = 12

QR² + PR² = 9² + 12² = 225 = 15² = PQ²

∴ ΔQPR is a right triangle

By comparing the ratio of the sides, we have;

cos(θ) = PR/PQ = 12/15 = 4/5, θ = cos⁻¹(4/5) = 36.9°

∠RPQ = 36.9°

sin(θ) = QR/PQ = 9/15 = 3/5

Similarly in triangle ΔABC, we have;

cos(θ) = BC/AB = 8/10 = 4/5

∠CBA = 36.9°

Therefore, ∠CBA ≅ ∠RPQ = 36.9°

Also ∠PRQ ≅ ∠BCA = 90° (Angle opposite hypotenuse side of right triangle

Therefore, ΔABC and ΔQPR are similar triangles by the Angle-Angle (AA) similarity theorem of two triangles.

. Use the quadratic formula to solve each quadratic real equation. Round
your answers to two decimal places. If there is no real solution, say so.
a) x^2 - 5x + 11 = 0
b) -2x^2 - 7x + 15 = 0
c) 4x^2 - 44x + 121 = 0​

Answers

Answer:

A. No real solution

B. 5 and -1.5

C. 5.5

Step-by-step explanation:

The quadratic formula is:

[tex]\begin{array}{*{20}c} {\frac{{ - b \pm \sqrt {b^2 - 4ac} }}{{2a}}} \end{array}[/tex], with a being the x² term, b being the x term, and c being the constant.

Let's solve for a.

[tex]\begin{array}{*{20}c} {\frac{{ 5 \pm \sqrt {5^2 - 4\cdot1\cdot11} }}{{2\cdot1}}} \end{array}[/tex]

[tex]\begin{array}{*{20}c} {\frac{{ 5 \pm \sqrt {25 - 44} }}{{2}}} \end{array}[/tex]

[tex]\begin{array}{*{20}c} {\frac{{ 5 \pm \sqrt {-19} }}{{2}}} \end{array}[/tex]

We can't take the square root of a negative number, so A has no real solution.

Let's do B now.

[tex]\begin{array}{*{20}c} {\frac{{ 7 \pm \sqrt {7^2 - 4\cdot-2\cdot15} }}{{2\cdot-2}}} \end{array}[/tex]

[tex]\begin{array}{*{20}c} {\frac{{ 7 \pm \sqrt {49 + 120} }}{{-4}}} \end{array}[/tex]

[tex]\begin{array}{*{20}c} {\frac{{ 7 \pm \sqrt {169} }}{{-4}}} \end{array}[/tex]

[tex]\begin{array}{*{20}c} {\frac{{ 7 \pm 13 }}{{-4}}} \end{array}[/tex]

[tex]\frac{7+13}{4} = 5\\\frac{7-13}{4}=-1.5[/tex]

So B has two solutions of 5 and -1.5.

Now to C!

[tex]\begin{array}{*{20}c} {\frac{{ -(-44) \pm \sqrt {-44^2 - 4\cdot4\cdot121} }}{{2\cdot4}}} \end{array}[/tex]

[tex]\begin{array}{*{20}c} {\frac{{ 44 \pm \sqrt {1936 - 1936} }}{{8}}} \end{array}[/tex]

[tex]\begin{array}{*{20}c} {\frac{{ 44 \pm 0}}{{8}}} \end{array}[/tex]

[tex]\frac{44}{8} = 5.5[/tex]

So c has one solution: 5.5

Hope this helped (and I'm sorry I'm late!)

a second degree equation in one variable example how many solutions does it have ?a second degree equation in one variable example how many solutions does it have ? is it possible to have many solutions or no solutions tions give an example for each

Answers

Answer:

  0, 1, or 2 real solutions

Step-by-step explanation:

Including complex and repeated solutions, a polynomial with real coefficients, and of degree n, always has n solutions.

If you're only concerned about real solutions, a 2nd degree equation in one variable may have 0, 1, or 2 real solutions. Here are some examples.

0 solutions: x^2 +1 = 01 solution: x^2 = 02 solutions: x^2 -1 = 0

What is this used for and how do i use it..?

Answers

you have to solve each one to get your answer and I think that your answer will be inside the circle

Answer:

This is called the Unit Circle. It is used in trigonometry. It had a radius of 1.

It helps you when using the trig function of sin cos and tan.

Hope this helps!!!!

Step-by-step explanation:

What is m Round the answer to the nearest whole number.
O 30°
O 35°
O 55°
O 60°

Answers

Answer:

30

Step-by-step explanation:

fufyfuf7fjcjcufuy7fufucyyxyvkbuvufudydy shut up

Which data set matches the box-and-whisker plot?
A) 12 13 15 19 23 23 25 26.5 28 30
B) 15 13 19 21 23 24 27 29 32
C) 11 31 13 15 19 21 21 25 27 29 31
D) 11 13 15 19 23 23 24 26.5 28 33​

Answers

Answer:

D) 11 13 15 19 23 23 24 26.5 28 33​

Step-by-step explanation:

The box-and-whisker plot displayed above has the following key values that we can use to identify which of the given data set it matches. It has:

Minimum value = 11

Q1 = 15

Median = 23

Q3 = 26

Maximum value = 33

From the options given, using just the max and min value, we can conclude that the data set in option D matches the box plot.

The data set in option D has a minimum value of 11, and a maximum value of 33.

What is the slope of the line containing the midpoint of the segment with endpoints at (2, 4) and (0, -2) and the midpoint of the segment with endpoints at (5, 1) and (1, 5)?Express your answer in simplest form. Plzzzz help!!!!

Answers

Answer:

slope = 1

Step-by-step explanation:

midpoint of (2, 4) and (0, -2)

(2 + 0)/2 = 1 and (4 + -2)/2 = 1

(1, 1)

midpoint of (5, 1) and (1, 5)

(5 + 1)/2 = 3 and (1 + 5)/2 = 3

(3, 3)

slope = (3-1)/(3-1) = 2/2 = 1

For each of the following paralellogram calculate the unknown angles marked. x, y and z

Answers

Answer:

x = 50°, y = z = 40°

Step-by-step explanation:

x = 50° ( Alternate angle )

z = 180° - (110 + 30)° = 180° - 140° = 40° ( sum of angles in Δ )

y = z = 40° ( Alternate angles )

Please answer quickly!

Answers

Answer:

T'(-1, -3)

U'(-8, -3)

V'(-9, -10)

W(1, -10)

Step-by-step explanation:

For each point, draw a segment from it to the line y = -x and perpendicular to the line, and extend it the same distance to the other side of the line.

T'(-1, -3)

U'(-8, -3)

V'(-9, -10)

W(1, -10)

Find the value of x.
76

Answers

This picture is depicting a straight line, therefore all we have to do is subtract 76 from 180. You get 104

Your answer is 104 degree

Henry gathered data about the types of nuts in five handfuls of mixed nuts. The data he gathered is shown in the table. Select the points that represent this data.

Answers

Answer:

Look below.

Step-by-step explanation:

The location of the coordinate plane will be shown in the graph.

What is coordinate geometry?

Coordinate geometry is the study of geometry using the points in space.

Henry gathered data about the types of nuts in five handfuls of mixed nuts.

The data he gathered is shown in the table.

Handful     Number of peanuts       Number of other nuts

  A                           9                                        7

  B                           6                                        5

  C                           8                                        9

  D                           5                                        7

  E                            7                                        4

The graph is shown below.

More about the coordinate geometry link is given below.

https://brainly.com/question/1601567

#SPJ2

**Yoxelt buys 4 1/2 gallons of soda. One-fourth of the soda he bought was Pepsi and the rest was Sprite. How many gallons of Pepsi did Yoxelt buy? Show all work below.

Answers

Answer:

  1 1/8

Step-by-step explanation:

1/4 of the 4 1/2 gallons were Pepsi, so the amount is ...

  (1/4)(9/2) = (1·9)/(4·2) = 9/8 = 1 1/8

Yoxelt bought 1 1/8 gallons of Pepsi.

When dividing polynomials using factorization, canceling identical factors in the denominator and the numerator will give the _______.

Answers

Answer:

quotient

Step-by-step explanation:

Answer:

quotient

Step-by-step explanation:

Cancelling identical factors in the numerator and the denominator will give the quotient. For example,x2+5x+6x+3=(x+2)(x+3)x+3 = x + 3

 

1. The cost of buying some books is partly constant and partly varies with the number of books bought. The cost is #4800 when 20 books are bought and #8000 when 40 are bought. Find the cost when 1000 books are bought

Answers

Answer:

Step-by-step explanation:

let the cost based on number of book bought be x and the constant be c:

4800 = 20x + c

8000 = 40x + c

c is common in both equations:

c =4800-20x

c = 8000-40x

equate the two:

4800-20x = 8000 - 40x

20x = 3200

x = 160

and c = 4800-20*160

c = 1600

Cost of 1000 books:

160*1000 + 1600

= 161600

-7p+2(5p-8)=6(p+6)-7

Answers

Answer:

-15

Step-by-step explanation:

-7p+10p-16=6p+36-7

3p-16=6p+29

3p-6p=29+16

-3p=45

p=45/-3

p=-15

P=5

-7p+2(5p-8)=6(p+6)-7
Distribute
-7p+10p-16=6p+36-7
Add -7p and 10p together
3p-16=6p+36-7
Subtract 7 from 36
3p-16=6p+29
+6p (on both sides)
9p-16=29
+16 (on both sides)
9p=45
Divide both sides by 9
P=5

Which expressions are equivalent to 2(b+3c)2(b+3c)2, left parenthesis, b, plus, 3, c, right parenthesis ?

Choose all answers that apply:

Choose all answers that apply:


(Choice A)

A

3(b+2c)3(b+2c)3, left parenthesis, b, plus, 2, c, right parenthesis


(Choice B)

B

(b+3c)+(b+3c)(b+3c)+(b+3c)left parenthesis, b, plus, 3, c, right parenthesis, plus, left parenthesis, b, plus, 3, c, right parenthesis


(Choice C)

C

2(b)+2(3c)2(b)+2(3c)2,

Answers

Answer:

B. (b+3c)+(b+3c) C. 2(b)+2(3c)

Step-by-step explanation:

Given this expression 2(b+3c), its equivalent expression is derived by simply opening up the bracket as shown below;

Open the parenthesis by multiplying the constant outside the bracket with all the variables in parenthesis.

= 2(b+3c)

=  2(b)+ 2(3c)

= 2b +2*3*c

= 2b +6c

It can also be written as sum of b+3c in 2 places i.e (b+3c)+(b+3c) because multiplying the function b+3c by 2 means we are to add the function by itself in two places.

Hence the equivalent expression are (b+3c)+(b+3c) and 2(b)+2(3c) or 2b+6c

An octagonal pyramid ... how many faces are there, how many vertices and how many edges? A triangular prism ... how many faces are there, how many vertices and how many edges? a triangular pyramid ... how many faces are there, how many vertices and how many edges?

Answers

1: 8 faces and 9 with the base 9 vertices and 16 edges

2: 3 faces and 5 with the bases 6 vertices and 9 edges

3: 3 faces and 4 with the base 4 vertices and 6 edges

1: 8 faces and 9 with the base 9 vertices and 16 edges

2: 3 faces and 5 with the bases 6 vertices and 9 edges

3: 3 faces and 4 with the base 4 vertices and 6 edges

The dosage for a certain drug calls for 20mg per kg per day and is divided into two doses(1every 12 hours) if a person weighs 197 pounds how much of the drug should be given each dose

Answers

Answer:

893.42

Step-by-step explanation:

1kg=2.205pounds

so 20mg is for 2.205pounds

therefore for 197pounds will be 1784.84mg

but the dose is once every 12hrs meaning twice a day so divide 1784.84/2 to get the pounds

write a equation of a line that has a slope of 5 and passes through the point (2,13)

Answers

Answer:

y = 5x + 3

Step-by-step explanation:

The equation of a line in slope- intercept form is

y = mx + c ( m is the slope and c the y- intercept )

Here m = 5, thus

y = 5x + c ← is the partial equation

To find c substitute (2, 13) into the partial equation

13 = 10 + c ⇒ c = 13 - 10 = 3

y = 5x + 3 ← equation of line

Helppp meeeew pleaseeeee

Answers

Answer:

Hey there!

1 1/10=1.1

2/25=0.08

Each serving requires 0.08 kg, and he has 1.1 kg.

Thus, he can make 1.1/0.08, or 13.75 servings.

Let me know if this helps :)

Answer:

13 servings of tofu dish.

Step-by-step explanation:

First, convert the fractions to decimal numbers:

1 1/10 kg = 0.1 kg

2/25 kg = 0.08 kg

Now find how many servings of tofu dish will cover 0.1 kg of tofu:

2/25 kg = 1 serving

1 1/10 kg = ?

= 1 1/10 ÷ 2/25 kg

= 11/10 × 25/2

= 55/4

= 13.75 servings

Approximate it to a whole number:

13 servings.

Is the product of two irrational numbers always an irrational​ number?

Answers

Answer:

Step-by-step explanation:

Not always.

√3 * √27 = √81 = 9

Which of the following symbols could correctly finish the statement. Select all that apply. 0___-8 = ≠ > < ≥ ≤

Answers

Answer:

>

Step-by-step explanation:

Even though its 0 its still greater than any negative number.

Answer:

Step-by-step explanation:

1. Find the greatest common divisor of the term 144x3y2and 81xy4​

Answers

Answer:

[tex]1296x^3y^4[/tex]

Step-by-step explanation:

Given the terms:

[tex]144x^3y^2[/tex]

and [tex]81xy^4[/tex]

To find:

Greatest Common Divisor of the two terms or Least Common Multiple (LCM) of two numbers = ?

Solution:

First of all, let us find the HCF (Highest Common Factor) for both the terms.

i.e. the terms which are common to both.

Let us factorize them.

[tex]144x^3y^2 = \underline{3 \times 3} \times 16\times \underline x \times x^{2}\times \underline{y^{2} }[/tex]

[tex]81xy^4= \underline {3\times 3}\times 9 \times \underline{x} \times \underline{y^2}\times y^2[/tex]

Common terms are underlined.

So, HCF of the terms = [tex]9xy^2[/tex]

Now, we know the property that product of two numbers is equal to the product of the numbers themselves.

HCF [tex]\times[/tex] LCM = [tex]144x^3y^2[/tex] [tex]\times[/tex] [tex]81xy^4[/tex]

[tex]LCM = \dfrac{144x^3y^2 \times 81xy^4}{9xy^2}\\\Rightarrow LCM = 144x^3y^2 \times 9x^{1-1}y^{4-2}\\\Rightarrow LCM = 144x^3y^2 \times 9x^{0}y^{2}\\\Rightarrow LCM = \bold{1296x^3y^4 }[/tex]

Wait times at a dentist's office are typically 21 minutes, with a standard deviation of 2 minutes. What percentage of people should be seen by the doctor between 17 and 25 minutes for this to be considered a normal distribution?

Answers

ANSWER: 95%

HOW:
95% of a group of data with a normal distribution is between two standard deviations to the right and left

Answer:

95%

Step by step explanation:

z = 17-21 / 2 and z = 25-21/2

z=-2 (2.28%) z=2 (97.72%)

97.72 - 2.28 = 5.44

100% - 5.44% is about equal to 95%

Carey earns $9.75 working part time on weekends. The table below shows the amount, a, Carey earns for working h hours. Carey’s Earnings h 0 1 3 a $0 $9.75 ? Which value completes the table to show the amount Carey earns for working 3 hours?

Answers

Answer:

$29.25

Step-by-step explanation:

For every 1 hour, Carey earns $9.75.  Multiply $9.75 by 3 to find out how much she earns for 3 hours of work.

$9.75 × 3 = $29.25

Carey earns $29.25 for working 3 hours.

Answer:

29.25

Step-by-step explanation:

I got it right on edge!! trust me

Anna's back Garden consists of a rectangular lawn measuring 9m by 7m, surrounded by a gravel path of width X metres. Find, and simplify, an expression for the total area of the garden.​

Answers

A rectangular lawn measuring 8m by 4m is surrounded by a flower bed of uniform width.

The combined area of the lawn and the flower bed is 165m^2. What is the width of the flower

:

Let x = the width of flower bed

:

Then the overall dimensions (flower bed & lawn) will be:

(2x + 8) by (2x + 4)

:

Overall area

(2x+8)*(2x+4) = 165

FOIL

4x^2 + 8x + 16x + 32 = 165

A quadratic equation

 

4x^2 + 24x + 32 - 165 = 0

4x^2 + 24x - 132 = 0

Simplify, divide by 4, results:

x^2 + 6x - 33 = 0

Use the quadratic formula to solve this

Given the equation y = 2x + 3 what is the slope?
x
3
2
idk

Answers

Answer:

The slope is 2

Step-by-step explanation:

Equation of a line is y = mx + c

where

m is the slope

c is the y intercept

From the question the equation is

y = 2x + 3

Comparing this equation with the general equation above

Slope / m = 2

Hope this helps you

Other Questions
What is chromosomes?Write in brief about the structure of a chromosome.must include all points with explanations All the edges of a cube have the same length. Tony claims that the formula SA = 6s, where s is the length ofeach side of the cube, can be used to calculate the surface area of a cube.a. Draw the net of a cube to determine if Tony's formula is correct.b. Why does this formula work for cubes?Frances believes this formula can be applied to calculate the surface area of any rectangular prism. Isshe correct? Why or why not?d. Using the dimensions of Length, Width and Height, create a formula that could be used to calculate thesurface area of any rectangular prism, and prove your formula by calculating the surface area of arectangular prism with dimensions L = 5m, W = 6m and H=8m. which of the following would differ if you compared the same reaction taking place with and without an enzymea. the chemical energy of the reactants b. the chemical change of the productc. the energy required to the reaction What two numbers multiply to negative 12 and add up to negative 13 PLZ HELP ASAP!!!! THANK YOU The disturbance that occurs as longitudional waves travel through a medium can be described as a series of A:oscillations and refractions B:propagations and compressions C:destructions and constructions D:rarefactions and compressions You have just purchased a motherboard that has an LGA 1156 socket for an Intel Pentium processor. What type of memory modules will you most likely need for this motherboard? A. DIP B. SIMM C. DIMM D. SODIMM Mary deeds her property to her sister for 15 years, in the form of a life estate. In addition, Mary also specifies that her sister's children will receive ownership of the property after the 15 years is up. Mary's sister's children are considered what? he says,"i am a businessman."change it into indirect Jose Suarez has been hired as sales manager at a new firm and is trying to come up with a sales force compensation method. He would like to have selling expenses relate directly to sales resources, an aggressive sales force, and minimization of non-selling tasks. What compensation method(s) would best fulfill his requirements plz translate to English and tell the language name The development manager is required to choose between two projects. Project A has an IRR of 25% and project B has an IRR of 30%. Which of the following statements is correct? A. If she can invest only in one project, the manager will choose project B B. None of the statements above is correct C. If she can invest only in one project, the manager will choose project A D. If she can invest in both projects, the manager will choose both projects A and B has a target debtequity ratio of 1.35. Its WACC is 8.3 percent, and the tax rate is 35 percent. If the companys cost of equity is 14 percent, what is its pretax cost of debt? (Do not round intermediate calculations. Enter yo Archie Co. purchased a framing machine for $60,000 on January 1, 2021. The machine is expected to have a four-year life, with a residual value of $5,000 at the end of four years. Using the sum-of-the years'-digits method, depreciation for 2022 and book value at December 31, 2022, would be: (Do not round intermediate calculations.) What is the reduced syllabus of ICSE class 9 Bernita and Derek each plot a number on a number line. the numbers are unique but have the same absolute value. the sum of the absolute values of the numbers is 50. what are the two numbers A. ASAB. CPCTCC. AASD. SAS Read the excerpt from Warriors Don't Cry. The announcer said it was 7:55 as Mama squeezed into a parking space, and we settled ourselves quietly for a moment, trying to identify the buzzing noise that seemed as if it were all around us. It resembled the sound of crowds at my high school football games. But how could that be A recent study of the relationship between social activity and education for a sample of corporate executives showed the following results. Social Activity Education Above Average Average Below Average College 30 20 10 High School 20 40 90 Grade School 10 50 130 Using 0.05 as the significance level, what is the critical value for the test statistic Trevor Company discloses supplementary operating segment information for its three reportable segments. Data for 20X8 are available as follows: Segment A Segment B Segment CSales $500,000 $300,000 $200,000Traceable operating expenses 250,000 120,000 90,000Allocable costs for the year was $180,000. Allocable costs are assigned based on the ratio of a segment's income before allocable costs to total income before allocable costs. The 20X8 operating profit for Segment B was:a. $180,000 b. $120,000 c. $126,000 d. $110,000 5. A Reforma Religiosa, movimento ocorrido na Europa no sculo XVI provocou: a) a extino da Companhia de Jesus. b) a ruptura da unidade da Igreja Catlica. c) a transferncia da sede do papado em Roma. d) o reconhecimento do islamismo como seita Crist.